Download as pdf or txt
Download as pdf or txt
You are on page 1of 13

Engineering Physics

(BPHY101L)

Dr. B. Ajitha
Assistant Professor
Division of Physics
VIT University
Chennai, India
ajitha.b@vit.ac.in
Problems BPHY101L
Dr. Ajitha

1) A particle is confined to a box of length L. It is described by the


wave function Ѱ = A sin(πx/L) for 0< X< L and Ѱ=0 everywhere
else. Calculate A?
We know that in case of a particle enclosed in a box of length L and
moving along X-axis, the wave function is given by ,
 n x 
  Asin   (1)
 L 
Where n is positive integer.
x
Given that,   Asin   (2)
 L 
Comparing eq. (2), with eq. (1), we get n=1.
Applying the normalization condition
L
 0
 *dx  1 we have,
L x 
 dx  1
2 2
A sin  
0  L 
2
x 
Problems
L

BPHY101L
A2 sin 2   dx  1
0  L  Dr. Ajitha

L  1  cos  2 x / L  
  dx  1
2
or A 
0
 2 

A2  L L 2 x 
2  0
dx   0
cos
L
dx   1

A2 L
2
 L  0  1 or A2
2
1

2
A 
2
L

2
A
L
3
Problems BPHY101L
Dr. Ajitha

2) A particle is moving in 1-D potential box (of infinite height) of


width 25 Å. Calculate the probability of finding the particle
within an interval of 5 Å at the centre of the box when it is in its
state of least energy?
We know that the wave function of a 2  n x 
particle enclosed within an infinite   x   sin 
potential barrier is given by, a  a 

When the particle is the least 2 x


  x  sin 
energy n=1, hence in this case, a  a 
At the centre of the box x=a/2, the probability of finding the particle in the unit
interval at the centre of box is given by
2
 2   (a / 2)   2 2 2
  x    sin 
2
   sin 
 a  a   a 2 a
The probability P in the interval Δx 2
P    x   x   x
2
is given by, a
4
Problems BPHY101L
Dr. Ajitha

According to the problem, a=25 Å=25×10-10 m and


Δx=5 Å =5×10-10 m

2
P    x   x 
2
 x
a
10
2  5 10
P 10
25 10

Probability = P  0.4

5
Problems BPHY101L
Dr. Ajitha

3) Find the least energy of an electron moving in the dimension in


as infinitely high potential box of width 1 Å, given mass of the
electron 9.11×10-31 kg and h=6.63×10-34 J-s?
We know that the values of energy of a
particle of mass ‘m’ moving in an infinite 1-D n2 h2
potential well of ‘a’ is given by,
En  2
(n  1, 2,3....)
8ma
The least energy of the particle can be h2
obtained by substituting n=1. The energy E1 
8ma 2
is given by

 6.63  10 
2
34
According to the given problem, E1  J
8  9.11 10 10 
m= 9.11×10-31 kg, a=1 Å=10-10 m 31 10
2
and h=6.63×10-34 J-s
9.1 1019
 9.1 1019 J  19
eV
1.602  10
 5.68 eV
6
Problems BPHY101L
Dr. Ajitha

4) An electron is bound by a potential which closely approaches in


infinite square well of width 2.5×10-10 m. Calculate the lowest
three permissible quantum energies the electron can have?

In case of infinite potential well, n2 h2


En 
8ma 2
Here, h=6.63×10-34 J-s, m= 9.11×10-31 kg,
 6.63  10 
2
a=2.5×10-10 m, Now: 34

E1  J
8  9.11 10  2.5  10 
2
31 10

9.63  1019
 9.63  1019 J  19
eV
1.602  10
E1  6 eV
Again, for second (n=2) and third E2  n 2 E1  4  6  24 eV
(3) quantum energies,
E3  n 2 E1  9  6  54 eV

7
Problems BPHY101L
Dr. Ajitha

5) An electron is confined to move two rigid walls separated by


10-9 m. Find the de-Broglie wavelength representing the first
three allowed energy states of the electron and corresponding
energies?
When electron moves to and fro b/w the rigid walls, it forms stationary wave
pattern with nodes at the walls.
So the distance ‘a’ b/w the walls must be a whole multiple of the de-Broglie half
wavelengths. Hence,


a  n   , Where n  1, 2,3....
2
2a
  Given that a  109 m  10 Å
n
2  10 Å
  20 Å ,10 Å,6.67 Å, ..... (if n  1, 2,3,...)
n

8
Problems BPHY101L
Dr. Ajitha

n2 h2
En  2
(n  1, 2,3....)
8ma
Here,h=6.63×10-34 J-s,
m= 9.11×10-31 kg,
 
2
34
a=10-9 m n 6.63  10
2

E1  J
  
2
8 9.11 1031 109

6.04  1020
 6.04  1020 n 2 J  n 2 eV
1.602  1019
 0.38 n 2 eV
For n=1, 2 and 3, we have

E1 = 0.38 eV, E2 = 1.52 eV and E3= 3.42 eV

9
Problems BPHY101L
Dr. Ajitha

6) An electron is bound by a potential which closely approaches an


infinite square well of width 2.5×10-10 m. Calculate the lowest
three permissible quantum energies the electron can have?
n2 h2
Given data: a= 2.5×10-10 m En 
8ma 2

 6.63  10 
2
34

E1  J
8  9.11 10  2.5  10 
2
31 10

 9.63  1019 J
9.63  1019
 19
eV
1.602  10
E1  6 eV
E2  24 eV
E3  54 eV
10
Problems BPHY101L
Dr. Ajitha

7) Find the energy of an electron moving in 1-D in an infinity high


potential box of width 1 Å, given mass of the electron 9.11×10-31
kgm and h=6.63×10-34 J-s?
h2
Given data: a=1 Å =10-10 m E
m= 9.11×10-31 kg & 8ma 2

 6.63  10 
h=6.63×10-34 J-s 34
2

E J
8  9.11 10 10 
2
31 10

 9.1 1019 J
9.1 1019
 19
eV
1.602  10

 0.58 eV

11
Problems BPHY101L
Dr. Ajitha

8) A particle is confined to 1-D infinite potential well of width


0.2×10-19 m. It is found that when the energy of the particle is
230 eV, its Eigen function has 5 antinodes. Find the mass of the
particle and show that it can never have energy equal to 1 keV?
It is obvious from the problem that the particle is in quantum state with n=5.
If E5 and E1 represent the energies in state 5 and state 1, respectively, then
E5  52 E1  230 eV
 230  1.602  1019 J
E5230  1.602  1019
or E1  2   14.7  1019 J
5 25
We knowthat
h2 h2
E1  2
 m
8ma 8E1a 2
Here, h=6.63×10-34 J-s, E1= 14.7×10-19 J and a=0.2×10-9 m
12
h2
Problems m
8 E1a 2 Here, h=6.63×10-34 J-s, E1= 14.7×10-19 J and a=0.2×10-9 m
BPHY101L
Dr. Ajitha

 6.63  10 
2
34

m J
8 14.7  10  0.2  10 
2
19 9

 9.31031 kg
When En= 1 keV =103 eV, n should be such that
En 103 1.602 1019
n  2

E1 14.7 1019
n 2 108.7 or
n  10.4

As n = 10.4 is not an integer,


En = 1 keV is not permitted value of energy

13

You might also like